LSAT and Law School Admissions Forum

Get expert LSAT preparation and law school admissions advice from PowerScore Test Preparation.

 Johnclem
  • Posts: 122
  • Joined: Dec 31, 2015
|
#27075
Hi powerscore,

For question15 , why is E wrong ? - I thought it may be wrong because the there's no assumption that's being disputed, as we see its stated in the first sentence by the backers of the program that this saving program is needed . ( so if its stated it can't be an assumption ) what do you think ?

Thanks
John
 Adam Tyson
PowerScore Staff
  • PowerScore Staff
  • Posts: 5153
  • Joined: Apr 14, 2011
|
#27076
Hey John, thanks for the question. I agree that the author is not disputing an assumption of the opposing argument, but is rather disputing a premise of that argument (hence answer choice A is correct). E seems to me to be wrong for that reason, but also for two other reasons: 1) I'm not so sure that the proponents of the proposed legislation make any assumption that such a program is needed. They say more savings is needed and they suggest this program to fill that need, but they may not assume that such a program is the only way to fill that need. Looked at conditionally, the program may be sufficient to solve the problem, but it may not be necessary to solve it. 2) Our author may actually agree with that assumption, if indeed it is an assumption made by the proponents. His disagreement is not with the idea that we need a program, but with the idea that this particular program will accomplish what it is intended to accomplish.

I hope that helps! Good analysis on your part!
 Wing yan
  • Posts: 4
  • Joined: Jan 05, 2017
|
#31879
Hi Powerscore,

I have question about the correct answer choice of this question. I have checked my PowerScore LAST Logical Reasoning:Question Type Training 1, the correct answer is B not A??????

Could you explain further why the correct answer should be B not E, I choose E?

Thanks!

Wing Yan
 Adam Tyson
PowerScore Staff
  • PowerScore Staff
  • Posts: 5153
  • Joined: Apr 14, 2011
|
#31895
Hey there Wing Yan, thanks for asking, but I am not seeing what you are seeing. In my copy of that book, the question is in Chapter 9 on Method of Reasoning questions, #26 on page 205. In the answer key on page 399, we show A as the correct answer. What are you seeing differently? Maybe you are looking at the answer key for a different chapter?

As discussed previously in this thread, answer E is wrong because the author does not dispute that a program is needed. Instead, he disputes that the proposed program will accomplish the goal it is intended to accomplish. He may agree that a program is needed, while claiming that this particular program won't do the job.

I hope that helps clear it up. If you have a copy of the book that shows an incorrect answer in the answer key, please let us know more about it so we can track it down and issue a correction.

Thanks!
 Wing yan
  • Posts: 4
  • Joined: Jan 05, 2017
|
#31902
Hi Adam, thanks for the reply, I'm talking about this question below, my Powerscore LSAT LR: Question Type training Volumn 1, 2016 Edition, P.34 Question No. 45, the answer key at the back is B, but I choose E, Could you explain why E is wrong, thanks!

(copy of question deleted due to LSAC copyright)
 Adam Tyson
PowerScore Staff
  • PowerScore Staff
  • Posts: 5153
  • Joined: Apr 14, 2011
|
#31907
Got it! What you are looking at there is question 14 from that test, rather than question 15, the subject of this thread. This is one of those LSAT stimuli that is followed by two question stems, something we don't see much of any more, if ever. For the correct thread for question 14, with additional explanation, go here:

lsat/viewtopic.php?t=8442

Answer E is much too strong! We know from the stimulus that our author thinks that this proposal will not work, because it will not stimulate additional savings, just a shift of savings from one place to another. Answer E says that no proposal could work, and that's a lot more than what we can take away from the stimulus. Just because one thing fails doesn't mean everything will fail!

Answer B is exactly what we said our author was saying - this proposal will not work to increase total savings, so of course any increase in savings (none) won't make up for the lost tax revenues (some).

Thanks for following up! I hope that makes sense. Most extreme answers (all, none, best, etc.) to Must Be True questions tend to be wrong, because we rarely have extreme evidence in the stimulus to support such extreme answers. Be suspicious of that kind of language when reviewing MBT answer choices.
 biskam
  • Posts: 124
  • Joined: Aug 18, 2017
|
#40447
Want to be clear here: the premise referenced in choice A is that the program will increase levels of personal savings in accounts?
 Claire Horan
PowerScore Staff
  • PowerScore Staff
  • Posts: 408
  • Joined: Apr 18, 2016
|
#42197
Specifically, the premise being challenged is that the proposal's implementation "would increase the amount of money available for banks to loan at a relatively small cost to the government in lost tax revenues."
 sherrilynm
  • Posts: 27
  • Joined: Mar 26, 2018
|
#44595
Can someone explain why C is incorrect? Doesn't the economist do exactly what C says - demonstrate that the implementation of this proposal is not feasible, because they've tried unsuccessfully in the past? What am I missing here?
 Adam Tyson
PowerScore Staff
  • PowerScore Staff
  • Posts: 5153
  • Joined: Apr 14, 2011
|
#44613
It's not that the program's implementation is not feasible, sherrilynm. That would mean that the proposal cannot be implemented, that it would be impossible to do it. That's not what our author is saying at all! Sure, we could do what the proposal recommends that we do. The issue is not feasibility, but effect - the author argues that the proposal, if implemented, will not have the intended effect, because similar proposals in the past did not have that effect.

So, we could give special tax incentives to people who put money into special accounts. No problem, totally feasible. Will more money be deposited into savings accounts, making that money available to banks to loan to others? Nope, not according to our author. The proposal could be followed, but we wouldn't get the intended result.

Read "feasible" as "possible to do it", rather than "it worked as intended", and you'll see the difference.

Get the most out of your LSAT Prep Plus subscription.

Analyze and track your performance with our Testing and Analytics Package.